If Jefferson is assigned to area 2, then which one of the following must be true?

sprozes on December 14, 2019

Please explain

Please explain why A is the correct answer choice?

Reply
Create a free account to read and take part in forum discussions.

Already have an account? log in

Skylar on December 14, 2019

@sprozes, thanks for your question.

Based on Rule 1, we know that M is in area 3. Based on this question, we know that we need to put J in area 2. This gives us:
1:
2: J
3: M

Now, let's look at O, since it has significant restrictions. Rule 2 tells us that O cannot go in area 1, so it can either go in area 2 or in area 3. Let's look at what happens if we put O in area 2:
1:
2: JO
3: M
Because of Rule 4, this means that we need to also put K in area 2. Since the game tells us that we can put no more than three rangers in a single area, area 2 is full. Now, we need to put P in area 3 because Rule 2 says P cannot go in area 1 and area 2 is full. We also need to put L in area three because Rule 3 says L needs to go with either K or M, and the area that K is in is full. This gives us:
1:
2: JOK
3: MPL
However, this is an invalid scenario because there are no rangers in area 1 and the game dictates that there must be at least one ranger in each area. Therefore, O cannot go in area 2.

So, O must be in area 3. This changes our setup to:
1:
2: J
3: MO

Let's consider which variables can go in area 1. J, M, and O can't, as they're already placed elsewhere. P can't because of Rule 2. This leaves us with only L or K.

When L is placed in area 1:
1: L
2: J
3: MO
Rule 3 dictates that L is either with M or K, and since L is in area 1 and M is in area 3, we must place K in area 1 to satisfy this rule. We are left with P, which can be placed in either area 2 or area 3. Our final setup is:
1: LK
2: J (possibly P)
3: MO (possibly P)

When K is placed in area 1:
1: K
2: J
3: MO
Rule 3 says that L must be with either K or M, so we have the option of putting L in area 1 or in area 3. Similarly, we have the option of putting P in either area 2 or area 3 (though we cannot put both P and L in area 3, as it would be past full). This gives us:
1: K (possibly L)
2: J (possibly P)
3: MO (possibly L) (possibly P)

Now, let's evaluate each answer choice. Remember, we are asked which MUST always be true, not whIch could be true.
(A) is correct because we see that K is in area 1 in each of our final two scenarios.
(B) is incorrect because L could be in area 3 in our second scenario.
(C) is incorrect because O is always in area 3.
(D) is incorrect because P could be in area 3 in both of our scenarios.
(E) is incorrect because P could be in area 2 in both of our scenarios.

Does that make sense? Please reach out with any other questions and best of luck with your studies!